Computing $mathbb{E}[X_{1} mid X_{1} + X_{2} + cdots + X_{n} = x]$ if $X_{1}, ldots X_{N}$ are i.i.d












0












$begingroup$


I would like to compute $mathbb{E}[X_{1} mid X_{1} + cdots X_{n} = x]$ if $X_{1}, ldots, X_{n}$ are i.i.d random variables.



Intuitively, I think that the answer should be $x/n$ because there are $n$ random variables with the same distribution. If they each contribute the same amount, then we would have $x/n cdot n = x$, which is what we want the sum to equal.



How can I show this, though?










share|cite|improve this question









$endgroup$












  • $begingroup$
    Presumably $E[sum_k X_k | sum_k X_k = x] = x$, assuming the event is not a null event.
    $endgroup$
    – copper.hat
    Dec 4 '18 at 18:58












  • $begingroup$
    Yes, that is correct. @copper.hat
    $endgroup$
    – joseph
    Dec 4 '18 at 18:58










  • $begingroup$
    Is $p[sum_k X_k=x] > 0$?
    $endgroup$
    – copper.hat
    Dec 4 '18 at 18:59










  • $begingroup$
    I don't know. I'm not given any information other than $X_{1}, ldots X_{n}$ being i.i.d.
    $endgroup$
    – joseph
    Dec 4 '18 at 19:00










  • $begingroup$
    The keyword here is linear. Expectation is linear, even when it is conditional. That is how you "show this". Either you already have a theorem which says as much, or the point of this exercise is to prove a special case of that theorem.
    $endgroup$
    – Arthur
    Dec 4 '18 at 19:05


















0












$begingroup$


I would like to compute $mathbb{E}[X_{1} mid X_{1} + cdots X_{n} = x]$ if $X_{1}, ldots, X_{n}$ are i.i.d random variables.



Intuitively, I think that the answer should be $x/n$ because there are $n$ random variables with the same distribution. If they each contribute the same amount, then we would have $x/n cdot n = x$, which is what we want the sum to equal.



How can I show this, though?










share|cite|improve this question









$endgroup$












  • $begingroup$
    Presumably $E[sum_k X_k | sum_k X_k = x] = x$, assuming the event is not a null event.
    $endgroup$
    – copper.hat
    Dec 4 '18 at 18:58












  • $begingroup$
    Yes, that is correct. @copper.hat
    $endgroup$
    – joseph
    Dec 4 '18 at 18:58










  • $begingroup$
    Is $p[sum_k X_k=x] > 0$?
    $endgroup$
    – copper.hat
    Dec 4 '18 at 18:59










  • $begingroup$
    I don't know. I'm not given any information other than $X_{1}, ldots X_{n}$ being i.i.d.
    $endgroup$
    – joseph
    Dec 4 '18 at 19:00










  • $begingroup$
    The keyword here is linear. Expectation is linear, even when it is conditional. That is how you "show this". Either you already have a theorem which says as much, or the point of this exercise is to prove a special case of that theorem.
    $endgroup$
    – Arthur
    Dec 4 '18 at 19:05
















0












0








0





$begingroup$


I would like to compute $mathbb{E}[X_{1} mid X_{1} + cdots X_{n} = x]$ if $X_{1}, ldots, X_{n}$ are i.i.d random variables.



Intuitively, I think that the answer should be $x/n$ because there are $n$ random variables with the same distribution. If they each contribute the same amount, then we would have $x/n cdot n = x$, which is what we want the sum to equal.



How can I show this, though?










share|cite|improve this question









$endgroup$




I would like to compute $mathbb{E}[X_{1} mid X_{1} + cdots X_{n} = x]$ if $X_{1}, ldots, X_{n}$ are i.i.d random variables.



Intuitively, I think that the answer should be $x/n$ because there are $n$ random variables with the same distribution. If they each contribute the same amount, then we would have $x/n cdot n = x$, which is what we want the sum to equal.



How can I show this, though?







probability probability-theory expected-value






share|cite|improve this question













share|cite|improve this question











share|cite|improve this question




share|cite|improve this question










asked Dec 4 '18 at 18:54









josephjoseph

510111




510111












  • $begingroup$
    Presumably $E[sum_k X_k | sum_k X_k = x] = x$, assuming the event is not a null event.
    $endgroup$
    – copper.hat
    Dec 4 '18 at 18:58












  • $begingroup$
    Yes, that is correct. @copper.hat
    $endgroup$
    – joseph
    Dec 4 '18 at 18:58










  • $begingroup$
    Is $p[sum_k X_k=x] > 0$?
    $endgroup$
    – copper.hat
    Dec 4 '18 at 18:59










  • $begingroup$
    I don't know. I'm not given any information other than $X_{1}, ldots X_{n}$ being i.i.d.
    $endgroup$
    – joseph
    Dec 4 '18 at 19:00










  • $begingroup$
    The keyword here is linear. Expectation is linear, even when it is conditional. That is how you "show this". Either you already have a theorem which says as much, or the point of this exercise is to prove a special case of that theorem.
    $endgroup$
    – Arthur
    Dec 4 '18 at 19:05




















  • $begingroup$
    Presumably $E[sum_k X_k | sum_k X_k = x] = x$, assuming the event is not a null event.
    $endgroup$
    – copper.hat
    Dec 4 '18 at 18:58












  • $begingroup$
    Yes, that is correct. @copper.hat
    $endgroup$
    – joseph
    Dec 4 '18 at 18:58










  • $begingroup$
    Is $p[sum_k X_k=x] > 0$?
    $endgroup$
    – copper.hat
    Dec 4 '18 at 18:59










  • $begingroup$
    I don't know. I'm not given any information other than $X_{1}, ldots X_{n}$ being i.i.d.
    $endgroup$
    – joseph
    Dec 4 '18 at 19:00










  • $begingroup$
    The keyword here is linear. Expectation is linear, even when it is conditional. That is how you "show this". Either you already have a theorem which says as much, or the point of this exercise is to prove a special case of that theorem.
    $endgroup$
    – Arthur
    Dec 4 '18 at 19:05


















$begingroup$
Presumably $E[sum_k X_k | sum_k X_k = x] = x$, assuming the event is not a null event.
$endgroup$
– copper.hat
Dec 4 '18 at 18:58






$begingroup$
Presumably $E[sum_k X_k | sum_k X_k = x] = x$, assuming the event is not a null event.
$endgroup$
– copper.hat
Dec 4 '18 at 18:58














$begingroup$
Yes, that is correct. @copper.hat
$endgroup$
– joseph
Dec 4 '18 at 18:58




$begingroup$
Yes, that is correct. @copper.hat
$endgroup$
– joseph
Dec 4 '18 at 18:58












$begingroup$
Is $p[sum_k X_k=x] > 0$?
$endgroup$
– copper.hat
Dec 4 '18 at 18:59




$begingroup$
Is $p[sum_k X_k=x] > 0$?
$endgroup$
– copper.hat
Dec 4 '18 at 18:59












$begingroup$
I don't know. I'm not given any information other than $X_{1}, ldots X_{n}$ being i.i.d.
$endgroup$
– joseph
Dec 4 '18 at 19:00




$begingroup$
I don't know. I'm not given any information other than $X_{1}, ldots X_{n}$ being i.i.d.
$endgroup$
– joseph
Dec 4 '18 at 19:00












$begingroup$
The keyword here is linear. Expectation is linear, even when it is conditional. That is how you "show this". Either you already have a theorem which says as much, or the point of this exercise is to prove a special case of that theorem.
$endgroup$
– Arthur
Dec 4 '18 at 19:05






$begingroup$
The keyword here is linear. Expectation is linear, even when it is conditional. That is how you "show this". Either you already have a theorem which says as much, or the point of this exercise is to prove a special case of that theorem.
$endgroup$
– Arthur
Dec 4 '18 at 19:05












1 Answer
1






active

oldest

votes


















3












$begingroup$

Your idea is exactly right. To express it rigorously, let $S_n=X_1+...+X_n$. The key observation is the following



Claim: $E(X_1|S_n)=E(X_i|S_n)$ for any $i$.



To see this, let $sigma$ be any permutation of the indices $1,dots, n$. Let $tilde{X_i}=X_{sigma(i)}$ and let $tilde{S_n}=sum_i tilde{X}_i$. Note that the random vector $(X_1,dots, X_n)$ has the same distribution as the random vector $(tilde{X_1}, dots, tilde{X_n})$ (by iid-ness). By applying the measurable function $(x_1, dots,x_n)to (x_1, sum_ix_i)$ to each of these random vectors,we conclude that $(X_1,S_n)=^d(tilde{X}_1,tilde{S_n})$. In particular, $E(X_1|S_n)=E(tilde{X}_1|tilde{S_n})$. On the other hand, $S_n=tilde{S_n}$ and $tilde{X}_1=X_{sigma(1)}$. So $E(X_1|S_n)=E(X_{sigma(1)}|S_n)$. But $sigma$ is arbitrary, so $sigma(1)$ could be any index $i$. This establishes the claim.



The rest of your argument goes through essentially unchanged. Noting that$sum_i E(X_i|S_n)=E(S_n|S_n)=S_n$,and applying the Claim, we get $nE(X_1|S_n)=S_n$ as desired.






share|cite|improve this answer









$endgroup$













    Your Answer





    StackExchange.ifUsing("editor", function () {
    return StackExchange.using("mathjaxEditing", function () {
    StackExchange.MarkdownEditor.creationCallbacks.add(function (editor, postfix) {
    StackExchange.mathjaxEditing.prepareWmdForMathJax(editor, postfix, [["$", "$"], ["\\(","\\)"]]);
    });
    });
    }, "mathjax-editing");

    StackExchange.ready(function() {
    var channelOptions = {
    tags: "".split(" "),
    id: "69"
    };
    initTagRenderer("".split(" "), "".split(" "), channelOptions);

    StackExchange.using("externalEditor", function() {
    // Have to fire editor after snippets, if snippets enabled
    if (StackExchange.settings.snippets.snippetsEnabled) {
    StackExchange.using("snippets", function() {
    createEditor();
    });
    }
    else {
    createEditor();
    }
    });

    function createEditor() {
    StackExchange.prepareEditor({
    heartbeatType: 'answer',
    autoActivateHeartbeat: false,
    convertImagesToLinks: true,
    noModals: true,
    showLowRepImageUploadWarning: true,
    reputationToPostImages: 10,
    bindNavPrevention: true,
    postfix: "",
    imageUploader: {
    brandingHtml: "Powered by u003ca class="icon-imgur-white" href="https://imgur.com/"u003eu003c/au003e",
    contentPolicyHtml: "User contributions licensed under u003ca href="https://creativecommons.org/licenses/by-sa/3.0/"u003ecc by-sa 3.0 with attribution requiredu003c/au003e u003ca href="https://stackoverflow.com/legal/content-policy"u003e(content policy)u003c/au003e",
    allowUrls: true
    },
    noCode: true, onDemand: true,
    discardSelector: ".discard-answer"
    ,immediatelyShowMarkdownHelp:true
    });


    }
    });














    draft saved

    draft discarded


















    StackExchange.ready(
    function () {
    StackExchange.openid.initPostLogin('.new-post-login', 'https%3a%2f%2fmath.stackexchange.com%2fquestions%2f3025963%2fcomputing-mathbbex-1-mid-x-1-x-2-cdots-x-n-x-if-x-1%23new-answer', 'question_page');
    }
    );

    Post as a guest















    Required, but never shown

























    1 Answer
    1






    active

    oldest

    votes








    1 Answer
    1






    active

    oldest

    votes









    active

    oldest

    votes






    active

    oldest

    votes









    3












    $begingroup$

    Your idea is exactly right. To express it rigorously, let $S_n=X_1+...+X_n$. The key observation is the following



    Claim: $E(X_1|S_n)=E(X_i|S_n)$ for any $i$.



    To see this, let $sigma$ be any permutation of the indices $1,dots, n$. Let $tilde{X_i}=X_{sigma(i)}$ and let $tilde{S_n}=sum_i tilde{X}_i$. Note that the random vector $(X_1,dots, X_n)$ has the same distribution as the random vector $(tilde{X_1}, dots, tilde{X_n})$ (by iid-ness). By applying the measurable function $(x_1, dots,x_n)to (x_1, sum_ix_i)$ to each of these random vectors,we conclude that $(X_1,S_n)=^d(tilde{X}_1,tilde{S_n})$. In particular, $E(X_1|S_n)=E(tilde{X}_1|tilde{S_n})$. On the other hand, $S_n=tilde{S_n}$ and $tilde{X}_1=X_{sigma(1)}$. So $E(X_1|S_n)=E(X_{sigma(1)}|S_n)$. But $sigma$ is arbitrary, so $sigma(1)$ could be any index $i$. This establishes the claim.



    The rest of your argument goes through essentially unchanged. Noting that$sum_i E(X_i|S_n)=E(S_n|S_n)=S_n$,and applying the Claim, we get $nE(X_1|S_n)=S_n$ as desired.






    share|cite|improve this answer









    $endgroup$


















      3












      $begingroup$

      Your idea is exactly right. To express it rigorously, let $S_n=X_1+...+X_n$. The key observation is the following



      Claim: $E(X_1|S_n)=E(X_i|S_n)$ for any $i$.



      To see this, let $sigma$ be any permutation of the indices $1,dots, n$. Let $tilde{X_i}=X_{sigma(i)}$ and let $tilde{S_n}=sum_i tilde{X}_i$. Note that the random vector $(X_1,dots, X_n)$ has the same distribution as the random vector $(tilde{X_1}, dots, tilde{X_n})$ (by iid-ness). By applying the measurable function $(x_1, dots,x_n)to (x_1, sum_ix_i)$ to each of these random vectors,we conclude that $(X_1,S_n)=^d(tilde{X}_1,tilde{S_n})$. In particular, $E(X_1|S_n)=E(tilde{X}_1|tilde{S_n})$. On the other hand, $S_n=tilde{S_n}$ and $tilde{X}_1=X_{sigma(1)}$. So $E(X_1|S_n)=E(X_{sigma(1)}|S_n)$. But $sigma$ is arbitrary, so $sigma(1)$ could be any index $i$. This establishes the claim.



      The rest of your argument goes through essentially unchanged. Noting that$sum_i E(X_i|S_n)=E(S_n|S_n)=S_n$,and applying the Claim, we get $nE(X_1|S_n)=S_n$ as desired.






      share|cite|improve this answer









      $endgroup$
















        3












        3








        3





        $begingroup$

        Your idea is exactly right. To express it rigorously, let $S_n=X_1+...+X_n$. The key observation is the following



        Claim: $E(X_1|S_n)=E(X_i|S_n)$ for any $i$.



        To see this, let $sigma$ be any permutation of the indices $1,dots, n$. Let $tilde{X_i}=X_{sigma(i)}$ and let $tilde{S_n}=sum_i tilde{X}_i$. Note that the random vector $(X_1,dots, X_n)$ has the same distribution as the random vector $(tilde{X_1}, dots, tilde{X_n})$ (by iid-ness). By applying the measurable function $(x_1, dots,x_n)to (x_1, sum_ix_i)$ to each of these random vectors,we conclude that $(X_1,S_n)=^d(tilde{X}_1,tilde{S_n})$. In particular, $E(X_1|S_n)=E(tilde{X}_1|tilde{S_n})$. On the other hand, $S_n=tilde{S_n}$ and $tilde{X}_1=X_{sigma(1)}$. So $E(X_1|S_n)=E(X_{sigma(1)}|S_n)$. But $sigma$ is arbitrary, so $sigma(1)$ could be any index $i$. This establishes the claim.



        The rest of your argument goes through essentially unchanged. Noting that$sum_i E(X_i|S_n)=E(S_n|S_n)=S_n$,and applying the Claim, we get $nE(X_1|S_n)=S_n$ as desired.






        share|cite|improve this answer









        $endgroup$



        Your idea is exactly right. To express it rigorously, let $S_n=X_1+...+X_n$. The key observation is the following



        Claim: $E(X_1|S_n)=E(X_i|S_n)$ for any $i$.



        To see this, let $sigma$ be any permutation of the indices $1,dots, n$. Let $tilde{X_i}=X_{sigma(i)}$ and let $tilde{S_n}=sum_i tilde{X}_i$. Note that the random vector $(X_1,dots, X_n)$ has the same distribution as the random vector $(tilde{X_1}, dots, tilde{X_n})$ (by iid-ness). By applying the measurable function $(x_1, dots,x_n)to (x_1, sum_ix_i)$ to each of these random vectors,we conclude that $(X_1,S_n)=^d(tilde{X}_1,tilde{S_n})$. In particular, $E(X_1|S_n)=E(tilde{X}_1|tilde{S_n})$. On the other hand, $S_n=tilde{S_n}$ and $tilde{X}_1=X_{sigma(1)}$. So $E(X_1|S_n)=E(X_{sigma(1)}|S_n)$. But $sigma$ is arbitrary, so $sigma(1)$ could be any index $i$. This establishes the claim.



        The rest of your argument goes through essentially unchanged. Noting that$sum_i E(X_i|S_n)=E(S_n|S_n)=S_n$,and applying the Claim, we get $nE(X_1|S_n)=S_n$ as desired.







        share|cite|improve this answer












        share|cite|improve this answer



        share|cite|improve this answer










        answered Dec 4 '18 at 19:15









        Mike HawkMike Hawk

        1,500110




        1,500110






























            draft saved

            draft discarded




















































            Thanks for contributing an answer to Mathematics Stack Exchange!


            • Please be sure to answer the question. Provide details and share your research!

            But avoid



            • Asking for help, clarification, or responding to other answers.

            • Making statements based on opinion; back them up with references or personal experience.


            Use MathJax to format equations. MathJax reference.


            To learn more, see our tips on writing great answers.




            draft saved


            draft discarded














            StackExchange.ready(
            function () {
            StackExchange.openid.initPostLogin('.new-post-login', 'https%3a%2f%2fmath.stackexchange.com%2fquestions%2f3025963%2fcomputing-mathbbex-1-mid-x-1-x-2-cdots-x-n-x-if-x-1%23new-answer', 'question_page');
            }
            );

            Post as a guest















            Required, but never shown





















































            Required, but never shown














            Required, but never shown












            Required, but never shown







            Required, but never shown

































            Required, but never shown














            Required, but never shown












            Required, but never shown







            Required, but never shown







            Popular posts from this blog

            Quarter-circle Tiles

            build a pushdown automaton that recognizes the reverse language of a given pushdown automaton?

            Mont Emei